You are on page 1of 31

Investment Management

Revision Workshop
Easter 2020

Eric Dyke
Question 1a
‘Pairs trading’ : Similar prices normally ( or correlated), buy cheap, sell dear 3
Short selling finances purchase / Profit fn includes P* 2
Xa =110 if Xb=80 /Profit always +ve /P*of 120 (50) …Profit 3,600 (1,500) /
Profit regardless 2

Explain the so called “pairs trading” rule in statistical arbitrage. Suppose a stock A is
currently valued at 80 and another stock B is valued at 110. How would you design a pairs
trading rule that generates profits when the two prices converge?
(7 marks)
The ‘pairs trading’ rule: Find two stocks that behave similarly in terms of prices. When
the two diverge, place a bet on convergence by buying the cheaper stock and shorting the
expensive stock.

Suppose two stocks, A and B, normally have similar prices, but currently A is trading at
80 and B is trading at 110. Buy xA units of A at a cost of 80xA financed by shorting xB units
of B giving proceeds of 110 𝑥𝐵 𝑠𝑢𝑐ℎ 𝑡ℎ𝑎𝑡 𝑥𝐴 𝑃𝐴 = 𝑃𝐵 𝑥𝐵
£2,500
110
If 𝑥𝐵 = 80, then 𝑥𝐴 = 80 = 110
80 £2,000

𝜋 = 𝑥𝐴 (𝑃∗ − 𝑃𝐴 ) + 𝑥𝐵 (𝑃𝐵 − 𝑃∗ ) £1,500

Profit
= 𝑃∗ 𝑥𝐴 − 𝑥𝐵 − 𝑥𝐴 𝑃𝐴 + 𝑥𝐵 𝑃𝐵 = 𝑃∗ (𝑥𝐴 − 𝑥𝐵 ) £1,000

If prices converge to 25, Profit = 25(110 – 80) = 750


£500

If prices converge to 50, Profit = 50(110 – 80) = 1,500 £0


Regardless, if the prices converge we make a trading profit. £20.0 £30.0 £40.0 £50.0
Price
£60.0 £70.0 £80.0
Question 1b
Sharpe 12.50% 3
Beta 0.80 3
Treynor 6.25% 3

A portfolio has total variance 16%, and an average return of 9%.


The market index has total variance 9% and an average return of
8%. The correlation between the return on the portfolio and the
return on the market index is 0.6.The risk free return is 4%. Work
out the Sharpe ratio and the Treynor ratio for the portfolio.
rP − rf 9%− 4% 5% (9 marks)
Sharpe ratio: = = = 12.50%
𝜎P 16% 0.40
𝜎𝑝𝑚
To calculate the Treynor ratio we need an expression for beta. 𝛽𝑃 = 𝜎 2
𝑚
𝜎pm
To evaluate the covariance we apply the given correlation. 𝜌pm =
𝜎p 𝜎m
𝜌pm 𝜎p 𝜎m 𝜌pm 𝜎p 0.6(0.4) Note: Assuming CAPM holds yields
∴ 𝛽𝑃 = 2 = = = 0.8 a beta of 1.2, but this is not utilising
𝜎𝑚 𝜎m 0.3
the specific information given.
Clearly CAPM does not hold.
rP− rf 9%−4%
Treynor ratio: = = 6.25%
𝛽P 0.80
Question 1c
Market risk 5.76% 3
Ideosyncratic risk 0.64% 3
Total risk 6.4% 3

Suppose you hold an equally weighted portfolio of 16 stocks. Each


stock has a beta of 0.8 and total variance 16%. The market index has
variance 9%. Assume the unsystematic risk is independent across the
stocks.

Calculate the total risk, market risk and unsystematic risk of the
portfolio.
2 2 2 2 2
(9 marks)
0.16 = 𝛽A 𝜎𝑚 + 𝜎𝑒,𝐴 = 0.8 0.09 + 𝜎𝑒,𝐴
2
∴ 𝜎𝑒,𝐴 = 0.16 − 0.0576 = 0.1024
2
1 16
𝜎𝑃2 = 𝛽𝐴2 𝜎𝑚
2 + 2
෍ 𝜎𝑒𝐴 = 0.82 0.09 + 2 0.1024
𝑛 16
𝑛=1
= 0.0576 + 0.0064 = 0.064

Note that the proportion of total risk that is idiosyncratic falls from
64% for one A share to 10% for 16 such shares.
Question 2a
Anti-takeover, protects management / detriment of
shareholders, dilution of bidder profit 4
Special rights issued, X-P, Flip-over on takeover 3

What is a poison-pill security? How does a flip-over plan work, and why can it be
called a poison-pill security?
7 marks)
Anti-takeover techniques can protect management to the detriment of shareholders by
diluting the gains to be had from a takeover. They do this by issuing special rights to
existing shareholders.

Preferred stock plan:


Target co issues a dividend in the form of convertible stock that is redeemable, in the
event of a takeover bid, for cash at the highest price of common stock in the last 12
months. A successful bidder must therefore pay the same high prices paid to gain control
for the preferred stock so issued.

Flip-over plan:
Here a dividend is awarded to buy common stock at an exercise price well above the
current price. In the event of a merger this option flips over and allows shareholders to
buy at a price X<P
Question 2b
Price 118.32 3
Duration 4.44 3
Convexity 24.09 (12.04) 3

A 5-year bond with annual coupons of 7% is trading on a yield-to-


maturity of 3%. What is the duration and the convexity of the bond?
𝑃=
7
+
7
+
7
++
7
+
107
= 118.32
(9 marks)
1.03 1.032 1.033 1.034 1.035
1 7 7 7 7 107
𝐷𝑢𝑟𝑎𝑡𝑖𝑜𝑛 = +2 + 3 + 4 + 5 = 4.44
118.32 1.03 1.032 1.033 1.034 1.035
4.442
𝑀𝑜𝑑𝑖𝑓𝑖𝑒𝑑 𝑑𝑢𝑟𝑎𝑡𝑖𝑜𝑛 = = 4.31
1.03
1 7 7 7 7 107
𝐶onvexity = 2 +6 + 12 + 20 + 30 = 24.09
118.32(1.03)2 1.03 1.032 1.033 1.034 1.035
Yield 3% P P' P"(1+r)^2 P"
Year Cash flow C/(1+r)^t tC/(1+r)^t t^2+t (t^2+t)(C/(1+r)^t)
1 7 £6.80 6.80 2 13.59 12.81
2 7 £6.60 13.20 6 39.59 37.32
3 7 £6.41 19.22 12 76.87 72.46
4 7 £6.22 24.88 20 124.39 117.25
5 107 £92.30 461.50 30 2,768.97 2,610.02
Price £118.32 525.58 3,023.42 2,849.86
Duration 4.442
MD 4.31
Convexity 24.09 In study guide 12.04
Question 2c
Gross cash flows -100k, 27k, 68k 3
Margin cash flows 30k, -3.6k, -26.4k 3
IRR -4.4 % 3

You long 1,000 units of a stock. At the end of the first year you
receive a dividend payment, and also sell 200 units of the stock after
the dividend transaction has cleared. On the remaining long position
you receive a further dividend at the end of the second year. You then
sell the remaining 800 units and the position is cleared. The prices
and dividend payments are given below.
Initial stock First year Price at the end Second year Price at the end
price dividend of year one dividend of year two
100 5 110 5 80

You make the transaction on the basis of a 70% initial and


maintenance margin requirement (implying you can borrow up to
30% of the required funds from the broker)

What is the two-year return on your long transaction? (9 marks)


Long position and margin requirement of 70%

0 1 2
Stock price £100.00 £110.00 £80.00
Shares bought (in each period) 1,000 -200 -800
Dividends per share £5.00 £5.00

Value of long position £100,000 £88,000


Max margin loan = (1-%) stock value £30,000 £26,400

0 1 2
Cash flow from share transactions -£100,000 £22,000 £64,000
Cash flow from dividends (+ve for long) £5,000 £4,000
Gross cash flow -£100,000 £27,000 £68,000
Margin cash flow £30,000 -£3,600 -£26,400
Net cash flow -£70,000 £23,400 £41,600

23,400 41,600
0 = −70,000 + + r = -4.40%
1+𝑟 (1 + 𝑟)2
Question 3a
Sharpe uses SD (reward to risk) 1
Agg returns are linear, but variance is overestimated 4
SD overestimated giving downward bias to SR 2

Explain why a measure of fund manager performance such as the Sharpe ratio controls
for the amount of risk in the fund’s investments. Why can the Sharpe ratio give a
misleading picture of the manager’s performance when the manager varies the level of
risk in the fund’s investments over time?
(7 marks)

(1) Sharpe ratio uses SD and can compare with the market.
(2) Average return is a linear average of the sub periods
(3) variance estimates put a disproportionally high weight on the high variance
regimes, thereby overestimating the aggregate variance

(4) Risk measurement is thereby biased upward and the Sharpe ratio becomes
downwardly biased
Question 3b
Liability value 4.33 3
Liability duration 2.90 3
Bond, cash holding 1.80 / 2.53 3

Consider a pension liability consisting of annual payments of $1m from year 1 through
to year 5. The spot interest rate is 5% (assume a flat term structure).

Outline the immunisation strategy of this pension liability against changes in the interest
rate by holding bonds with duration 7 years. (9 marks)

1 1
𝑉𝑎𝑙𝑢𝑒 𝑜𝑓 𝑙𝑖𝑎𝑏𝑖𝑙𝑖𝑡𝑦 = 1− = 4.33
0.05 1.055
1
1
1.051
+ 2 1
1.052
+3 1
1.053
+4 1
1.054
+5 1
1.055
Liability duration = 4.33
= 2.90

Duration Assets $mn Liabilities $mn Duration


7.00 Bond A x Pension liability 4.33 2.90
0.00 Cash / Bond B 4.33 - x

2.43 Total 4.33 Total 4.33 2.90

𝑥 4.33 − 𝑥 4.33
Require: 7+ 0= 2.90
4.33 4.33 4.33
∴ 𝑥 = 1.80 (and 2.53 in cash)
Question 3c
Factor 1 risk premium 3.02% 3
Factor 2 risk premium 2.74% 3
Portfolio return 8.71% 3

Suppose the stock market has a 2-factor structure. The risk free rate is 3% and the
factor-betas and expected returns on two well-diversified portfolios A and B are
given in the table below.

Portfolio Expected return Factor 1 beta Factor 2 beta


A 8.13% 0.7 1.1
B 10.90% 0.8 2

What is the expected return on a portfolio with factor 1 beta of 0.8 and factor
2 beta of 1.2?
(9 marks)
Arbitrage pricing theory Simultaneous equations
𝑟𝐴 = 𝑟𝑓 + 𝑏𝐴1 𝜆1 + 𝑏𝐴2 𝜆2 8.13% − 3% = 0.7𝜆1 + 1.1𝜆2
10.90% − 3% = 0.8𝜆1 + 2𝜆2
𝑟𝐵 = 𝑟𝑓 + 𝑏𝐵1 𝜆1 + 𝑏𝐵2 𝜆2

𝜆1 = 3.02% 𝑎𝑛𝑑 𝜆2 = 2.74%


𝑟𝐶 = 𝑟𝑓 + 0.8𝜆1 + 1.2𝜆2 = 8.71%
Question 4a
Myopia generally suboptimal 2
Implication of constant RRA 2
Condition for myopia to be optimal 3

Explain why asset allocation over longer time horizons can be approached as a
“myopic” problem when relative risk aversion is independent of wealth.
(7 marks)
Can investors afford to be 'myopic’, or must the whole investment horizon be
taken into account? The argument is that long–lived investors can optimally
bear more risk as they will experience up and down movements over a longer
time horizon than short–lived investors who invest only over one period.
Myopia is suboptimal.
If RRA is constant investors take the same relative amount of risk regardless of
wealth, so if they keep half their wealth risky they still keep half risky even if
they doubled their wealth.
Campbell and Viceira say myopic asset allocation is optimal for long–lived
investors if investors tend to invest the same fractions of their wealth risky and
risk free, and will therefore not need to take into account wealth effects from
one period to the other. It is only if there is a violation of this condition that
long–lived investors need to look beyond the next period when making their
asset allocation choices.
Question 4b
2-fund separation 2
Optimum formula, Weight 0.12 in market 5
Interpret result 2

The utility of an investor can be described by the following utility function, which depends on
the expected return µ, and the variance, 𝜎𝑃2 , of the investor’s portfolio: 𝑈(𝜇, 𝜎 2 ) = 𝜇 − 2𝜎𝑃2
Suppose all investments available to the investor are located on the security market line
cutting through the risk free return rf = 3% and the market portfolio with expected return =
15%. The variance of the market portfolio is 25%.
What portfolio would you recommend to this investor? (9 marks)

𝜇 = 𝑤𝑟𝑚 +(1 − 𝑤)𝑟𝑓 Two fund separation


2 +(1 − 𝑤)2 𝜎 2 + 2𝑤 1 − 𝑤 𝜌
𝜎𝑝2 = 𝑤 2 𝜎𝑚 𝑟𝑓
2 2
𝑚,𝑟𝑓 𝜎𝑚 𝜎𝑟𝑓 = 𝑤 𝜎𝑚

Expected utility of holding a portfolio is :

𝑈(𝜇, 𝜎 2 ) = 𝜇 − 2𝜎𝑃2
= 𝑤𝑟𝑚 + 1 − 𝑤 𝑟𝑓 − 2𝑤 2 𝜎𝑚
2
𝑑𝑈 2 =0
= 𝑟𝑚 − 𝑟𝑓 − 4𝑤𝜎𝑚
𝑑𝑤
(𝑟𝑚 − 𝑟𝑓 ) 12%
∴w = 2 = = 0.12
4𝜎m 4(25)%
Utility is maximised when 12% of funds are in
the market and 88% in the risk free asset
Question 4c
Weighting x and 1-x 3
Cov(rA, rM) + other Expressions 3
Solving using relationship 30.44% A / 69.56% B 3

If investors are variance-averse they hold the market portfolio, which predicts that
𝐸𝑟𝑖 − 𝑟𝐹 = 𝐶𝑜𝑛𝑠𝑡𝑎𝑛𝑡 ∗ 𝐶𝑜𝑣(𝑟𝑖 , 𝑟𝑀 ), where 𝐸𝑟𝑖 is the expected return on stock 𝑖; 𝑟𝐹 is
the risk free return; and 𝐶𝑜𝑣(𝑟𝑖 , 𝑟𝑀 ) is the covariance between the return of the stock
and the return on the market index.

Suppose the stock market consists of only 2 stocks A and B and a risk free asset. Let
𝐸𝑟𝐴 = 10%; 𝐸𝑟𝐵 = 8%; 𝑟𝐹 = 4%; the variance of A is 𝑉𝑎𝑟 𝑟𝐴 = 0.20; the variance of B
is 𝑉𝑎𝑟 𝑟𝐵 = 0.15; and the covariance between A and B is 𝐶𝑜𝑣 𝑟𝐴 , 𝑟𝐵 = 0.40.

What is the composition of the market index? (9 marks)

𝑟𝐴 − 𝑟𝑓 = 𝑐𝑜𝑛𝑠𝑡𝑎𝑛𝑡 ∗ 𝐶𝑜𝑣(𝑟𝐴 , 𝑟𝑚 ) for all securities


𝐶𝑜𝑣(𝑟𝐴 , 𝑟𝑚 ) = 𝐶𝑜𝑣(𝑟𝐴 , 𝑥𝑟𝐴 +(1 − 𝑥) 𝑟𝐵 ) = 𝑥𝑉𝑎𝑟𝐴 +(1 − 𝑥)𝐶𝑜𝑣(𝑟𝐴 , 𝑟𝐵 )
𝐶𝑜𝑣(𝑟𝐵 , 𝑟𝑚 ) = 𝐶𝑜𝑣(𝑟𝐵 , 𝑥𝑟𝐴 +(1 − 𝑥) 𝑟𝐵 ) = (1 − 𝑥)𝑉𝑎𝑟𝐵 +𝑥𝐶𝑜𝑣(𝑟𝐴 , 𝑟𝐵 )
𝑟𝐴 − 𝑟𝑓 𝐶𝑜𝑣(𝑟𝐴 , 𝑟𝑚 )
= 1.5 =
𝑟𝐵 − 𝑟𝑓 𝐶𝑜𝑣(𝑟𝐵 , 𝑟𝑚 )
𝑥𝑉𝑎𝑟𝐴 +(1−𝑥)𝐶𝑜𝑣(𝑟𝐴 , 𝑟𝐵 ) 0.20𝑥+0.4(1−𝑥)
∴ = = 1.5
(1−𝑥)𝑉𝑎𝑟𝐵 + 𝑥𝐶𝑜𝑣(𝑟𝐴 , 𝑟𝐵 ) 0.15(1−𝑥)+ 0.4𝑥
∴ 𝑥 = 0.3044 𝑜𝑟 30.44% 𝑖𝑛 𝐴 𝑎𝑛𝑑 69.56% 𝑖𝑛 𝐵
Question 5a
Why is it a puzzle? High return on equity, implausible risk aversion 3
Survivorship bias (too long) or falling real bond yields (too short) 4

Explain what we mean by the “equity premium puzzle”. Based on the subject guide, explain
one way to resolve this puzzle. (7 marks)
US 1926:2002 Geometric Standard
The puzzle is that the return on equity is much return deviation

greater than the return on bonds as might Large Co stock 10.01 % 20.55 %

reasonably be expected from relative risks. Long Treasury 5.68 % 8.24 %

An investor holding Treasury bonds and 𝜌 2


large company stocks must derive equal 𝐸(𝑢(𝑥)) = 𝜇 − 2 𝜎
utility from them. Using the utility 𝜌 𝜌
0.0568 − (0.0824) = 0.1001 − (0.2055)2
function from the study guide we can 2 2
work out the risk aversion coefficient, ρ ρ = 2.61
Fitting a more rigorous theoretical … an investor would be indifferent
model Mehra and Prescott calculate an between paying an insurance premium
implied risk aversion coefficient of and keeping 51 for sure, and a gamble
between 30 and 40. 30 implies where there is a 50/50 chance of
receiving 50 and of receiving 200.
Time series too long: Survivorship Time series too short: Evidence suggests
bias causes us to overestimate the that a large equity premium is a product
return to equity. of the period 1950-2000. Beyond that it
is much smaller.
Question 5b
Yields 6%, 5.04%, 4.03% 3
Spots 6%, 5%, 4% 3
Lending profitable NPV -£2.64 f23= 2.03% < 5% 3

Consider the data below on prices and coupon rates:


Bond price per £100 nominal Coupon rate Time to maturity
£99.06 5% 1 year
£105.51 8% 2 years
£94.38 2% 3 years

Calculate the yields y1 and y2, of the one and two-year bonds and
provide an expression for solving the three-year bond yield y3.
Calculate the one and two-year spot rates s1 and s2 and provide an
expression for solving the three-year spot rate s3. You are to assume
that coupons are paid annually, with the next payment being in 12
months’ time.

You get the opportunity to borrow £100,000 at the end of year 2,


with a repayment of £105,000 at the end of year 3. Should you take
this borrowing opportunity? Explain.
(9 marks)
105
Yields: 99.06 = 1+𝑦
1
→ 𝑦1 = 6%

8 8
105.51 = 1+𝑦 + (1+𝑦 2 → 𝑦2 = 5.04%
2 2)
2 2 102 You are not required to work
94.38 = 1+𝑦 + (1+𝑦 + (1+𝑦 → 𝑦3 = 4.03%
3 3 )2 3)
3 out this numerical answer.

105
Spots: 99.06 = 1+𝑠 → 𝑠1 = 6%
1
8 108
105.51 = 1.06 + (1+𝑠 2 → 𝑠2 = 5%
2)

2 2 102 You are not required to work


102.92 = + + → 𝑠3 = 4% out this numerical answer.
1.06 (1.05)2 (1+𝑠3 )3

102
90.68 =
(1 + 𝑠3 )3

102 But you will find it


(1 + 𝑠3 )3 = = 1.1248
90.68 useful to evaluate this.
It suffices to look directly at the NPV of the loan transaction:
100 105 100 105
NPV = (1+𝑠 − (1+𝑠 = − 1.1248 = £90.70 − £93.34 = −£2.64
2 )2 3 )3 1.052

£100 £105
Alternatively, demonstrate that £90.70
the borrowing opportunity LOAN: 𝑠2 = 5%
entails an arbitrage loss. 0 1 2 3
£93.34
£100 £105
Lend NPV £105 (thousand) for repayment end year 3
Borrow NPV£100 for repayment in year 2
No future liabilities, zero risk, immediate loss.

Alternatively show that the 5% rate on the loan exceeds the market fwd fwd rate
for the period
1.124864
(1 + s2 )2 (1 + 𝑓23 ) = (1 + s3 )3 ∴ 𝑓23 = − 1 = 2.03%
(1.05)2

The opportunity entails borrowing at 5%, exceeding the market forward rate.
It should not be taken.
Question 5c
Standardisation Z formula 3
Ln (VT/Vt) = -0.1611 30 day VaR 3
Ln (VT/Vt) = -0.2245 60 day VaR 3

Suppose the value of a portfolio over a 30-day period is log-normally distributed so that the
30-day log-return has mean 0.565% and volatility (i.e. standard deviation) 7.167%.

What is the 30-day 1% VaR of the portfolio, expressed as log-returns? Use the fact that for a
standard normal variable, the distribution function 𝑁(∙) yields 𝑁(−2.33) = 1%.

What is the 60-day 1% VaR?


(9 marks)

𝑉
𝑙𝑛 𝑉𝑇 − 𝜇30 𝑑𝑎𝑦
𝑡
𝑍= = - 2.33
𝜎30 𝑑𝑎𝑦

𝑉𝑇
𝑙𝑛 = 0.00565 - 2.33 (0.07167) = - 0.1611
𝑉𝑡

𝑉 𝑉
𝑙𝑛𝑉𝑇 − 𝜇60 𝑑𝑎𝑦 𝑙𝑛 𝑉𝑇 − 0.00565(2)
For a 60-day VaR, the expected ∴𝑍= 𝑡
= 𝑡
𝜎60 𝑑𝑎𝑦 (0.071672 (2))0.5
return and variance are both 𝑉
linear in time. 𝑙𝑛 𝑉𝑇 − 0.00565(2)
𝑡
= = - 2.33
0.07167 2
𝑉𝑇
∴ 𝑙𝑛 = 0.01130 - 2.33 (0.10136)= - 0.2245
𝑉𝑡
Question 6a
Small frequent gains (7% OTM and 3 m options) /
very large infrequent losses 4
Asymmetrical risk underestimated 2
SR overestimated 1

Explain how the strategy of selling deep out-of-the-money options can distort the
measurement of investment performance for hedge funds.
(7 marks)
1. If the put options have a strike price 7% below the current level, and the
maturity is less than three months, then most months, the puts expire still out
of the money, and the hedge funds simply receive a stable cash inflow.

2. In some months large index falls will push the puts into the money and the
hedge fund will potentially lose heavily.

3. Trading strategy involves asymmetrical risk: small but frequent gains are
measured against large but infrequent losses. Therefore, a statistical analysis
of the fund’s performance will underestimate the risk of the fund.

4. With the denominator underestimated the Sharpe ratio will be overstated.


Question 6b
Clean price 111.15 3
Dirty 115.27 3
Accrued 2.71 3

Consider a bond that pays an annual coupon of 6% of the bond’s face value of £100. The
number of days since the last coupon payment is 165 and the bond matures in 4 years and
200 days. The yield-to-maturity of the bond is 3%.
200 165
You are given the following numbers: 1.03 = 1.0163 𝑎𝑛𝑑 1.03 = 1.0135. What are 365 365

the dirty and the clean prices of the bond? What is the accrued interest?
(9 marks)
The dirty price is the discounted value of the coupon payments and the clean price now.
6 6 6 6 106 1 6 6 6 106
PD = 200 + 200 + 200 + 200 + 200 = 200 6+ + + +
1.031 1.032 1.033 1.034
1.03365 1.031+365 1.032+365 1.033+365 1.034+365 1.03365

6 + 𝑃𝐶 6 + 97.4229
= 200 = = 115.27
1.0163
1.03365
165
Approx accrued interest is 6*(days since last coupon/days between coupons) =6
365
= 2.71

Dirty price = Clean price + Accrued interest. ∴ Clean price = 115.27 – 2.71 = 112.56
Question 6c
Alpha 4.2% 3
Unsystematic 14.60% 3
Weights 44.37% in A and 55.63% in mkt 3
Consider the following table with data on some capital market investments:
Asset/portfolio Expected return Beta Total variance
Risk free asset 4%
Market index 12% 1 0.15
Portfolio A 13% 0.6 0.20

The Treynor-Black model states that if Portfolio A has a positive alpha you should
optimally mix it with the market index, the weight 𝑤 on A being
𝛼 𝛼 is Jensen’s alpha of Portfolio A;
𝑤=𝛼 𝛽𝐴 is the beta of portfolio A;
1−𝛽𝐴 + 𝐸𝑟𝑀 −𝑟𝐹 𝑉𝑎𝑟(𝑒𝐴 )/𝑉𝑎𝑟(𝑟𝑀 )
𝐸(𝑟𝑀 ) is the expected return on the market index;
𝑟𝐹 is the risk free rate;
𝑉𝑎𝑟(𝑒𝐴 ) is the idiosyncratic variance of Portfolio A;
𝑉𝑎𝑟(𝑟𝑀 ) is the variance of the market portfolio).

Work out the value of w. (9 marks)

Jensen’s alpha = 13% - (4% + 0.6*(12% - 4%)) = 4.2%.


Var(eA) = 0.20 – 0.6^2 0.15 = 0.1460.
w = 0.042/(0.042*(1-0.6) + (0.12-0.04)*(0.1460/0.15) = 0.4437. and 1-w = 0.5563

NB The optimum risky portfolio has a return of 12.44% and a variance of 13.02%
respectively. Its SR of 0.2340 exceeds that of the passive portfolio (0.2066) and the
active one (0.2012)
Question 7a
Both risk neutral/ distress costs 2
Illiquidity / Cash flow concerns / CAR 3
Liquidity / Capital loss concerns / VaR 2

Discuss similarities and differences of the risk management problems of a company


owning predominantly illiquid fixed assets such as a manufacturing firm, and a
company owning predominantly liquid financial assets such as a bank.
(7 marks)
Risk management for corporations and banks needs to be justified on grounds other
than through managing risk of their owners. Unlike shareholders they are risk averse
and manage risk in order to avoid the costs of financial distress.
For corporations these can occur when there are operational losses because earnings
are the main source of funds in a crisis. Losses can lead to a loss of valuable tax
deductions, costly renegotiations with suppliers, customers and banks, quite apart
from potential direct bankruptcy costs. To hedge against large losses corporations may
employ the technique of CaR – cash flow at risk.
Unlike corporations, many of the assets held by banks are highly liquid and can be
transformed to cash at short notice. Therefore banks can raise funds by liquidating
assets rather than relying on operating cash flows. Hence banks protect themselves
from the costs of financial distress by ensuring they have a sufficient buffer of equity
capital to cover any losses on the market value of portfolios. They employ the
technique of VaR – value at risk
Question 7b
Price to sell future money 96 (4.17% borrow rate) 3 You are given the following price data on European
Price to buy 94.44 (5.88% lend rate) 3 put and call options on a non-dividend paying stock.
Arbitrage Profit 1.556- 0.0190 =1.5365 3

X represents the option exercise price and the current stock price is 100. Work out whether
there are arbitrage opportunities in this market, and whether you could make money on a
transaction to buy $1m in one years’ time, simultaneously selling the same amount forward,
while paying a 0.1% transaction cost for each transaction? (9 marks)
𝑋 𝑋
Arbitrage free, require PV(X) = =𝑆+𝑃−𝐶 ∴ 𝑟𝑓 = −1
1+𝑟𝑓 𝑆+𝑃−𝐶
Call Put S X PV(X) rf 𝐷𝐹 (price of a future 1)
96 1
12 8 100 100 96 4.17% = 0.96 = 1.0417
100
18 3 100 90 85 5.88% 85 1
= 0.9444 = 1.0588
90

Difference per 100 = 1.556

Borrow at low rate and lend at high rate.


100 100
Present profit on trading future sum of 100 is − = 96 − 94.44 = 1.556
1.0417 1.0588

Net profit per 100 = 1.556 - 0.001 (96 + 85) = 1.556 – 0.0190 = 1.5365

Assuming rf = 0% and arbitraging both sides of put-call parity equation is incorrect.


Question8c
Question 7c
Roll
Treeformula -c^2
P(H|Ask) 0.55 P(H|Bid) 0.45 3
Spread 1
Ask 107.5 c = 0.12247 for A Spread 1.11% 3
Bid 102.5
Spread 2 c = 0.15811 for B Spread 2.64% 3

Suppose the auto-covariance of successive transaction price changes of stock A is


− 0.015 and the corresponding number for stock B is −0.025. The average price of stock
A during the estimation period is $22 and the corresponding number for stock B is $12.
Use Roll’s model to estimate the bid-ask spread (as % of the average stock price) for the
two stocks. (9 marks)

𝐶𝑜𝑣 Δ𝑝t−1 , Δ𝑝t = −𝑐 2

∴ −𝑐 2 = −0.015

𝑐= 0.015 = 0.12247
C is half the spread, so … 2𝑐 = 0.24495

2𝑐 2(0.0150.5 ) 0.24495
∴ 𝑥𝐴 = = = 22 =1.11%
𝑃 22

2𝑐 2(0.0250.5 )
∴ 𝑥𝐵 = 𝑃
= 12
= 2.64%
Question 8a
EMH 1
Momentum, Fads 2
Cognitive biases: slow reaction, weights recent, overconfidence, framing 4

Discuss the basis and the evidence for the criticism of the efficient market hypothesis
outlined in the area of behavioural finance.
(7 marks)
The EMH is about asset prices having a random walk property: that the expected future
discounted prices are just today’s prices. Impossible to predict future price deviations on
the basis of current information.
Empirical predictability in asset prices. Momentum. A study found that portfolios of the best-
performing stocks in the recent past (three- or 12-month holding period) tend to outperform
other stocks. Although the performance of individual stocks remains highly unpredictable.
Fads hypothesis asserts that the stock market overreacts to news, leading to positive
autocorrelation over shorter time horizons and a reversal or negative autocorrelation over
longer time horizons.

Behavioural finance provides a foundation for the momentum and reversal effects based
on imperfections in the human ability to process new information rationally. There is
substantial evidence to suggest that we are sometimes too slow to react to news (causing
momentum), that we tend to add too much weight to recent evidence (causing reversal
effects), that we tend to be overconfident and finally that our choices are affected by
framing.
Question 8b
Investment in rf / calls 951.23k / 48.77k 3
Put-call parity and # calls 98.84 3
Rf / index /puts 575.16k / 395.35k / 29.49k 3

You seek to build a 12-month, put-protected investment consisting of the stock market
index, 12-month put options on the index, and a risk free part. The following table contains
data on the index and 12-month put and call options on the index.
Current index level 4,000
12-month call payoff at maturity Max(0, Index level – 4,000)
12-month put payoff at maturity Max(0, 4,000 – Index level)
Risk free rate 5.13%
Current 12-month call price 493.44
Current 12-month put price 298.36

Your objective is to capture the maximum share of the return of the index when the
index has positive return, at the same time as securing zero return when the index has
negative return. Assuming you invest 1m, derive the breakdown of your investment.
(9 marks)
Call portfolio
1,000,000 1,000,000
∴ = = 951,203.27
1+𝑟𝑓 1.0513
Leaving, to invest in calls 48,796.73
1,000,000.00
1,000,000 − 951,203.27 4,000
𝑖. 𝑒. = 98.8909 𝑐𝑎𝑙𝑙𝑠 98.8909𝐶 = 98.8909 𝑆 + 𝑃 −
1+ 𝑟𝑓
4,000

Put portfolio
Invest portfolio 98.8909 (4,000) = 395,563.62
Money in puts 98.8909 (4,000) = 29,505.09
4,000
Invest risk free 951,229.15 – 98.8385 ( )= 574,941.86
1.0513
1,000,010.57

Check for a 10% rise in the index (and a 10% fall)


𝑆
Outcome = 574,941.86 (1.0513) + 98.8909 max 4,000 − 𝑆, 0 + 395,563.62 = 1,039,556.36
4,000
A 10% market rise will result in a rise in our put protected portfolio of 39.56%
Question8c
Question 8c
Tree P(H|Ask)
Tree P(H|Ask)0.55
0.561 P(H|Bid)
P(H|Bid)0.45
0.458 3
Ask
Ask 107.5
72.44 3
Bid
Bid102.5
68.31 3

A risk neutral market maker clears buy and sell orders for a share
in a perfectly competitive environment. She believes that there are
equal probabilities the share is worth 90 or 50. Traders are drawn
randomly from a pool of uninformed traders, who buy with a
probability of 40% and sell with a probability of 60%, and
informed traders who buy if the share is worth 90 and sell if the
share is worth 50. 10% of traders are informed and 90% are
uninformed. Work out the initial bid and ask prices.
(9 marks)
Always apply Bayes for Glosten Milgrom
𝐴𝑠𝑘 𝑝𝑟𝑖𝑐𝑒 = 𝑃 90 𝐴𝑠𝑘 90 + 𝑃 50 𝐴𝑠𝑘 50
Tree diagram of conditional = 0.561 90 + (0.439)50 = 72.44
probabilities
𝐵𝑖𝑑 𝑝𝑟𝑖𝑐𝑒 = 𝑃 90 𝐵𝑖𝑑 90 + 𝑃 50 𝐵𝑖𝑑 50
= 0.458 90 + (0.542)50 = 68.31

1
Buy at Ask 0.1
Bayes Law:
P 𝐴𝑠𝑘 90 = 0.46
If these are equal the two
I P 𝐴𝑠𝑘 90 P(90)
P 90 𝐴𝑠𝑘 = conditional probabilities
0.1 P(𝐴𝑠𝑘) are the same.

Buy at Ask 0.36


P 𝐴𝑠𝑘 90 P(90)
90 0.4 =
P 𝐴𝑠𝑘 90 P 90 + P 𝐴𝑠𝑘 50 P 50
0.9 N
P 90 = 0.5 0.46
0.6
= 0.46+0.36 = 0.561
Sell at Bid 0.54

P 𝐵𝑖𝑑 90 = 0.54 P 50 𝐴𝑠𝑘 = 1 − 0.561 = 0.439

P 𝐵𝑖𝑑 90 P(90)
P 90 𝐵𝑖𝑑 =
P(𝐵𝑖𝑑)
I P 𝐵𝑖𝑑 90 P(90)
P 50 = 0.5 0.1
1 Sell at Bid 0.1 =
P 𝐵𝑖𝑑 90 P 90 + P 𝐵𝑖𝑑 50 P 50
50 Buy at Ask 0.36
0.4 0.54
P 𝐴𝑠𝑘 50 = 0.36 =
0.54+0.64
= 0.458
0.9 N
0.6 Sell at Bid 0.54

P 𝐵𝑖𝑑 50 = 0.64 P 50 Bid = 1 − 0.458 = 0.542


30
End

You might also like